On note pour un entier \(n\) non-nul : \[S_n = \sum_{i=0}^{n-1} \sum_{j=0}^{n-1} \dfrac{1}{i+j+1} \textrm{ et } I_n = \int_0^n \Bigl(\int_0^n \dfrac{\mathrm{ \;d}y}{x+y+1} \Bigr) \mathrm{ \;d}x\]

  1. Calculer l’intégrale \(I_n\) pour \(n > 0\).

  2. Déterminer un équivalent de la suite \((I_n)\).

    On pourra utiliser un développement limité à l’ordre \(1\) de \(\ln\) en \(0\).
  3. Encadrer \(S_n\) à l’aide de \(I_n\) pour \(n > 0\).

  4. En déduire un équivalent de \((S_n)\).


Barre utilisateur

[ID: 1976] [Date de publication: 12 mai 2021 13:12] [Catégorie(s): Suites dont le terme général est défini par une intégrale ] [ Nombre commentaires: 1] [nombre d'éditeurs: 1 ] [Editeur(s): Emmanuel Vieillard-Baron ] [nombre d'auteurs: 3 ] [Auteur(s): Emmanuel Vieillard-Baron Alain Soyeur François Capaces ]




Solution(s)

Solution(s)

Exercice 92
Par Emmanuel Vieillard-Baron Alain Soyeur François Capaces le 12 mai 2021 13:12
  1. Une première intégration suivant la variable \(x\) conduit à \(I_n = \int_0^n \ln(x+n+1)\mathrm{ \;d}x - \int_0^n \ln(x+1)\mathrm{ \;d}x\). En intégrant par parties, on trouve finalement \(\boxed{I_n = (2n+1)\ln(2n+1) - 2(n+1)\ln(n+1)}\).

  2. Pour tout \(x\in\left]-1,+\infty\right[\), on a : \(\ln\left(1+x\right)=x+\underset{x \rightarrow 0}{o}\left(x\right)\) donc si \(\left(u_n\right)\) est une suite réelle convergeant vers \(0\) alors \(\ln\left(1+u_n\right)=u_n+\underset{n \rightarrow +\infty}{o}\left(u_n\right)\). Écrivons \[\begin{aligned} I_n &=& (2n+1)\ln(2n+1) - 2(n+1)\ln(n+1)\\ &=& \left(2n+1\right)\left(\ln\left(2n\right)+\ln\left(1+1/2n\right) \right) -2\left(n+1\right)\left(\ln n +\ln\left(1+1/n\right)\right) \\ &=& \left(2n+1\right)\left(\ln\left(2n\right)+\dfrac{1}{2n}+\underset{n \rightarrow +\infty}{o}\left(\dfrac{1}{2n}\right) \right) -2\left(n+1\right)\left(\ln n + \dfrac{1}{n}+\underset{n \rightarrow +\infty}{o}\left(\dfrac{1}{n}\right) \right) \\ &=&\left(2n+1\right)\ln 2 -\ln n -1 -\dfrac{3}{2n}+\underset{n \rightarrow +\infty}{o}\left(1\right)\\ &=& 2n\ln 2\underbrace{\left(1+\dfrac{ \ln 2 -\ln n -1 -{\scriptstyle 3\over\scriptstyle 2n} +\underset{n \rightarrow +\infty}{o}\left(1\right) }{2n \ln 2} \right)}_{\xrightarrow[n \rightarrow +\infty]{}1}\\ &\underset{n \rightarrow +\infty}{\sim}& \boxed{2n\ln 2}\end{aligned}\]

  3. Comme \(x \mapsto 1/x\) est décroissante, et que \[I_n = \sum_{i=0}^{n-1}\sum_{j=0}^{n-1} \int_{i}^{i+1}\int_{j}^{j+1} \dfrac{1}{x+y+1} \mathrm{ \;d}x\mathrm{ \;d}y\] on en tire l’encadrement \[\sum_{i=0}^{n-1}\sum_{j=0}^{n-1} \dfrac{1}{i+j+3}\leqslant I_n \leqslant S_n\] d’où \(I_n \leqslant S_n\) et en sortant les termes de la somme à gauche : \[I_n \leqslant S_n \leqslant I_n + \sum_{j=2}^{n+1}\dfrac{1}{j}\] Mais toujours en comparant cette somme avec une intégrale, \(\sum_{j=2}^{n+1} \dfrac{1}{j} \leqslant\int_1^{n+1}\dfrac{\mathrm{ \;d}t}{t}\). Finalement, \[I_n \leqslant S_n \leqslant I_n + \ln(n+1)\]

  4. On tire facilement des deux questions précédentes que \(\boxed{S_n \underset{n \rightarrow +\infty}{\sim} 2n\ln 2}\).


Documents à télécharger